Summing Up The Nearest!

Calculus Level 5

n = 1 1 n 4 7 \large \displaystyle \sum_{n=1}^{\infty} \frac{1}{\lfloor \sqrt[4]{n} \;\rceil ^7}

The above sum can be expressed as a b π α + c d π β \frac{a}{b}\pi^{\alpha} + \frac{c}{d}\pi^{\beta} , where a , b , c , d , α , a,b,c,d,\alpha , and β \beta are positive integers, with a a and b b being co-prime, c c and d d are co-prime.

Find a + b + c + d + α + β a+b+c+d+\alpha +\beta .

Details and Assumptions :

x \bullet \lfloor x \rceil denotes the nearest integer to x x .

Try my other calculus challenges here


The answer is 1003.

This section requires Javascript.
You are seeing this because something didn't load right. We suggest you, (a) try refreshing the page, (b) enabling javascript if it is disabled on your browser and, finally, (c) loading the non-javascript version of this page . We're sorry about the hassle.

2 solutions

Hasan Kassim
Jun 30, 2015

First note that for a certain integer k k :

x = { k if x [ k , k + 0.5 ) k + 1 if x ( k + 0.5 , k + 1 ] \displaystyle \lfloor x \rceil = \left\{ \begin{array}{lr} k & \text{if} \; x \in [k,k+0.5) \\k+1 & \text{if} \; x \in (k+0.5, k+1 ] \end{array} \right.

Let k n 4 < k + 1 \displaystyle k\leq\sqrt[4]{n} < k+1

Where k k is an integer ranging between 1 1 and \infty .

Let us split the interval into two :

k n 4 < k + 0.5 and k + 0.5 < n 4 < k + 1 \displaystyle k\leq \sqrt[4]{n} < k+0.5 \; \; \text{and} \; \; k+0.5 < \sqrt[4]{n} <k+1

Note that we restricted the value at k + 0.5 k+0.5 , since n 4 \sqrt[4]{n} is either irrational or integral. Now rewrite the intervals as:

k 4 n < ( k + 0.5 ) 4 and ( k + 0.5 ) 4 < n < ( k + 1 ) 4 \displaystyle k^4\leq n < (k+0.5)^4 \; \; \text{and} \; \; (k+0.5 )^4< n <(k+1 )^4

n n is an integer , so rewrite as:

k 4 n ( k + 0.5 ) 4 and ( k + 0.5 ) 4 n < ( k + 1 ) 4 \displaystyle k^4\leq n \leq \lfloor (k+0.5)^4 \rfloor \; \; \text{and} \; \; \lceil (k+0.5 )^4 \rceil \leq n <(k+1 )^4

Therefore our Sum can be written as :

S = n = 1 1 n 4 7 \displaystyle S= \sum_{n=1}^{\infty} \frac{1}{ \lfloor \sqrt[4]{n} \rceil ^7 }

= k = 1 ( n = k 4 ( k + 0.5 ) 4 1 k 7 + n = ( k + 0.5 ) 4 ( k + 1 ) 4 1 1 ( k + 1 ) 7 ) \displaystyle = \sum_{k=1}^{\infty} \bigg( \sum_{n=k^4}^{ \lfloor (k+0.5)^4 \rfloor} \frac{1}{k^7} + \sum_{n=\lceil (k+0.5 )^4 \rceil }^{(k+1)^4 -1} \frac{1}{(k+1)^7} \bigg)

The work inside the brackets is easy (as we have no n n ):

S = k = 1 ( ( k + 0.5 ) 4 k 4 + 1 k 7 + ( k + 1 ) 4 1 ( k + 0.5 ) 4 + 1 ( k + 1 ) 7 ) \displaystyle S= \sum_{k=1}^{\infty} \bigg( \frac{ \lfloor (k+0.5)^4 \rfloor - k^4 +1}{k^7} + \frac{(k+1)^4 -1 - \lceil (k+0.5 )^4 \rceil +1 }{(k+1)^7} \bigg)

Note that we should find an expression for ( k + 0.5 ) 4 \lfloor (k+0.5)^4 \rfloor :

( k + 0.5 ) 4 = k 4 + 2 k 3 + 3 2 k 2 + 1 2 k + 1 16 \displaystyle (k+0.5)^4 = k^4 + 2k^3 + \frac{3}{2} k^2 + \frac{1}{2} k +\frac{1}{16}

= k 4 + 2 k 3 + k ( 3 k + 1 ) 2 + 1 16 \displaystyle = k^4 + 2k^3 + \frac{k(3k+1)}{2} + \frac{1}{16}

For any integral k k , either k k or 3 k + 1 3k+1 will be even, hence :

k ( 3 k + 1 ) 2 is an integer \displaystyle \frac{k(3k+1)}{2} \;\;\;\;\text{is an integer}

And so :

( k + 0.5 ) 4 = ( k + 0.5 ) 4 1 16 \displaystyle \lfloor (k+0.5)^4 \rfloor = (k+0.5)^4 - \frac{1}{16}

Consequently:

( k + 0.5 ) 4 = ( k + 0.5 ) 4 1 16 + 1 \displaystyle \lceil (k+0.5 )^4 \rceil = (k+0.5)^4 - \frac{1}{16} +1

Now back to our sum:

S = k = 1 ( ( k + 0.5 ) 4 1 16 k 4 + 1 k 7 + ( k + 1 ) 4 1 ( k + 0.5 ) 4 + 1 16 ( k + 1 ) 7 ) \displaystyle S = \sum_{k=1}^{\infty} \bigg( \frac{ (k+0.5)^4 - \frac{1}{16} - k^4 +1}{k^7} + \frac{(k+1)^4 -1 - (k+0.5)^4 + \frac{1}{16} }{(k+1)^7} \bigg)

= k = 1 ( k + 0.5 ) 4 1 16 k 4 + 1 k 7 + k = 1 ( k + 1 ) 4 1 ( k + 0.5 ) 4 + 1 16 ( k + 1 ) 7 \displaystyle = \sum_{k=1}^{\infty} \frac{ (k+0.5)^4 - \frac{1}{16} - k^4 +1}{k^7} + \sum_{k=1}^{\infty} \frac{(k+1)^4 -1 - (k+0.5)^4 + \frac{1}{16} }{(k+1)^7}

Now shift the index of the second series by one:

= k = 1 ( k + 0.5 ) 4 1 16 k 4 + 1 k 7 + k = 2 k 4 1 ( k 0.5 ) 4 + 1 16 k 7 \displaystyle = \sum_{k=1}^{\infty} \frac{ (k+0.5)^4 - \frac{1}{16} - k^4 +1}{k^7} + \sum_{k=2}^{\infty} \frac{k^4 -1 - (k-0.5)^4 + \frac{1}{16} }{k^7}

Note that the second series is zero at k = 1 k=1 :

= k = 1 ( k + 0.5 ) 4 1 16 k 4 + 1 k 7 + k = 1 k 4 1 ( k 0.5 ) 4 + 1 16 k 7 \displaystyle = \sum_{k=1}^{\infty} \frac{ (k+0.5)^4 - \frac{1}{16} - k^4 +1}{k^7} + \sum_{k=1}^{\infty} \frac{k^4 -1 - (k-0.5)^4 + \frac{1}{16} }{k^7}

= k = 1 ( ( k + 0.5 ) 4 1 16 k 4 + 1 k 7 + k 4 1 ( k 0.5 ) 4 + 1 16 k 7 ) \displaystyle = \sum_{k=1}^{\infty} \bigg( \frac{ (k+0.5)^4 - \frac{1}{16} - k^4 +1}{k^7} + \frac{k^4 -1 - (k-0.5)^4 + \frac{1}{16} }{k^7} \bigg)

= k = 1 ( k + 0.5 ) 4 ( k 0.5 ) 4 k 7 \displaystyle = \sum_{k=1}^{\infty} \frac{ (k+0.5)^4 - (k-0.5)^4}{k^7}

= k = 1 4 k 3 + k k 7 \displaystyle = \sum_{k=1}^{\infty} \frac{ 4k^3 +k}{k^7}

= 4 k = 1 1 k 4 + k = 1 1 k 6 \displaystyle = 4\sum_{k=1}^{\infty} \frac{1}{k^4} + \sum_{k=1}^{\infty} \frac{1}{k^6}

= 4 ζ ( 4 ) + ζ ( 6 ) \displaystyle = 4\zeta (4) + \zeta (6)

S = 2 45 π 4 + 1 945 π 6 \displaystyle \boxed{ S = \frac{ 2}{45} \pi^4 + \frac{1}{945}\pi^6 }

Aman Rajput
Jul 9, 2015

I used the general expression of nint function.

S N ( s ) = n = 1 1 ( n 1 / N ) s \displaystyle S_N(s) = \sum_{n=1}^{\infty} \frac{1}{(\lfloor n^{1/N} \rceil)^s}

Expressions are available for s > 3 s>3

I also used . S 4 ( s ) = 4 ζ ( s 3 ) + ζ ( s 1 ) S_4(s) = 4\zeta(s-3) + \zeta(s-1)

For s = 7 , we have = 2 π 4 45 + π 6 945 \boxed{\frac{2\pi^4}{45} + \frac{\pi^6}{945}}

Thanks for sharing that! That's the first time I hear about the this Formula, do you think there exist a closed form for S N ( s ) S_N (s) ??.

Hasan Kassim - 5 years, 11 months ago

Log in to reply

Not a general closed form actually..

but it has closed forms for differnt N N and s s

and i love the way you present your solution.. U latex master :D !!!!

Aman Rajput - 5 years, 11 months ago

Log in to reply

And a genius like you,, may derive the closed form .

but i havent actually studied and read it anywhere yet. :) :) :)

Aman Rajput - 5 years, 11 months ago

Log in to reply

@Aman Rajput Thanks my friend ! I will Check it out sometime. :) :)

Hasan Kassim - 5 years, 11 months ago

0 pending reports

×

Problem Loading...

Note Loading...

Set Loading...